The conditions under which the Taylor polynomial $P_n(x)$ will converge to $f(x)$












0












$begingroup$


Background



It is well-known that for the following function:



$$f(x) = begin{cases}
e^{frac{-1}{x^2}} & text{if} & x neq 0 \
0 & text{if} & x = 0
end{cases} $$



the Taylor polynomial does not converge to $f(x)$ as $nto infty$, although $f^{(k)}(0)$ exists and it is 0 for all $kgeq 0$. The problem is with the general conditions under which a Taylor polynomial will converge to the function.



Problem



In general, what conditions exist (or do they exist at all) such that the Taylor polynomial of a function will converge to the function as $nto infty$?










share|cite|improve this question











$endgroup$












  • $begingroup$
    The $n$-th derivative of any polynomial function is another polynomial function and therefore a continuous function.
    $endgroup$
    – José Carlos Santos
    Dec 5 '18 at 11:03










  • $begingroup$
    @JoséCarlosSantos Thank you for your answer. I understand it now that my question should now change a bit.
    $endgroup$
    – hephaes
    Dec 5 '18 at 11:29
















0












$begingroup$


Background



It is well-known that for the following function:



$$f(x) = begin{cases}
e^{frac{-1}{x^2}} & text{if} & x neq 0 \
0 & text{if} & x = 0
end{cases} $$



the Taylor polynomial does not converge to $f(x)$ as $nto infty$, although $f^{(k)}(0)$ exists and it is 0 for all $kgeq 0$. The problem is with the general conditions under which a Taylor polynomial will converge to the function.



Problem



In general, what conditions exist (or do they exist at all) such that the Taylor polynomial of a function will converge to the function as $nto infty$?










share|cite|improve this question











$endgroup$












  • $begingroup$
    The $n$-th derivative of any polynomial function is another polynomial function and therefore a continuous function.
    $endgroup$
    – José Carlos Santos
    Dec 5 '18 at 11:03










  • $begingroup$
    @JoséCarlosSantos Thank you for your answer. I understand it now that my question should now change a bit.
    $endgroup$
    – hephaes
    Dec 5 '18 at 11:29














0












0








0





$begingroup$


Background



It is well-known that for the following function:



$$f(x) = begin{cases}
e^{frac{-1}{x^2}} & text{if} & x neq 0 \
0 & text{if} & x = 0
end{cases} $$



the Taylor polynomial does not converge to $f(x)$ as $nto infty$, although $f^{(k)}(0)$ exists and it is 0 for all $kgeq 0$. The problem is with the general conditions under which a Taylor polynomial will converge to the function.



Problem



In general, what conditions exist (or do they exist at all) such that the Taylor polynomial of a function will converge to the function as $nto infty$?










share|cite|improve this question











$endgroup$




Background



It is well-known that for the following function:



$$f(x) = begin{cases}
e^{frac{-1}{x^2}} & text{if} & x neq 0 \
0 & text{if} & x = 0
end{cases} $$



the Taylor polynomial does not converge to $f(x)$ as $nto infty$, although $f^{(k)}(0)$ exists and it is 0 for all $kgeq 0$. The problem is with the general conditions under which a Taylor polynomial will converge to the function.



Problem



In general, what conditions exist (or do they exist at all) such that the Taylor polynomial of a function will converge to the function as $nto infty$?







real-analysis analysis taylor-expansion exponential-function






share|cite|improve this question















share|cite|improve this question













share|cite|improve this question




share|cite|improve this question








edited Dec 5 '18 at 11:29







hephaes

















asked Dec 5 '18 at 11:01









hephaeshephaes

1709




1709












  • $begingroup$
    The $n$-th derivative of any polynomial function is another polynomial function and therefore a continuous function.
    $endgroup$
    – José Carlos Santos
    Dec 5 '18 at 11:03










  • $begingroup$
    @JoséCarlosSantos Thank you for your answer. I understand it now that my question should now change a bit.
    $endgroup$
    – hephaes
    Dec 5 '18 at 11:29


















  • $begingroup$
    The $n$-th derivative of any polynomial function is another polynomial function and therefore a continuous function.
    $endgroup$
    – José Carlos Santos
    Dec 5 '18 at 11:03










  • $begingroup$
    @JoséCarlosSantos Thank you for your answer. I understand it now that my question should now change a bit.
    $endgroup$
    – hephaes
    Dec 5 '18 at 11:29
















$begingroup$
The $n$-th derivative of any polynomial function is another polynomial function and therefore a continuous function.
$endgroup$
– José Carlos Santos
Dec 5 '18 at 11:03




$begingroup$
The $n$-th derivative of any polynomial function is another polynomial function and therefore a continuous function.
$endgroup$
– José Carlos Santos
Dec 5 '18 at 11:03












$begingroup$
@JoséCarlosSantos Thank you for your answer. I understand it now that my question should now change a bit.
$endgroup$
– hephaes
Dec 5 '18 at 11:29




$begingroup$
@JoséCarlosSantos Thank you for your answer. I understand it now that my question should now change a bit.
$endgroup$
– hephaes
Dec 5 '18 at 11:29










1 Answer
1






active

oldest

votes


















1












$begingroup$

One such condition is given by Bernstein's theorem: if $(forall xin D_f)(forall ninmathbb{N}):f^{(n)}(x)geqslant0$, then the Taylor series of $f$ converges pointwise to $f(x)$ in the neighborhood of every point of $D_f$.






share|cite|improve this answer









$endgroup$













    Your Answer





    StackExchange.ifUsing("editor", function () {
    return StackExchange.using("mathjaxEditing", function () {
    StackExchange.MarkdownEditor.creationCallbacks.add(function (editor, postfix) {
    StackExchange.mathjaxEditing.prepareWmdForMathJax(editor, postfix, [["$", "$"], ["\\(","\\)"]]);
    });
    });
    }, "mathjax-editing");

    StackExchange.ready(function() {
    var channelOptions = {
    tags: "".split(" "),
    id: "69"
    };
    initTagRenderer("".split(" "), "".split(" "), channelOptions);

    StackExchange.using("externalEditor", function() {
    // Have to fire editor after snippets, if snippets enabled
    if (StackExchange.settings.snippets.snippetsEnabled) {
    StackExchange.using("snippets", function() {
    createEditor();
    });
    }
    else {
    createEditor();
    }
    });

    function createEditor() {
    StackExchange.prepareEditor({
    heartbeatType: 'answer',
    autoActivateHeartbeat: false,
    convertImagesToLinks: true,
    noModals: true,
    showLowRepImageUploadWarning: true,
    reputationToPostImages: 10,
    bindNavPrevention: true,
    postfix: "",
    imageUploader: {
    brandingHtml: "Powered by u003ca class="icon-imgur-white" href="https://imgur.com/"u003eu003c/au003e",
    contentPolicyHtml: "User contributions licensed under u003ca href="https://creativecommons.org/licenses/by-sa/3.0/"u003ecc by-sa 3.0 with attribution requiredu003c/au003e u003ca href="https://stackoverflow.com/legal/content-policy"u003e(content policy)u003c/au003e",
    allowUrls: true
    },
    noCode: true, onDemand: true,
    discardSelector: ".discard-answer"
    ,immediatelyShowMarkdownHelp:true
    });


    }
    });














    draft saved

    draft discarded


















    StackExchange.ready(
    function () {
    StackExchange.openid.initPostLogin('.new-post-login', 'https%3a%2f%2fmath.stackexchange.com%2fquestions%2f3026940%2fthe-conditions-under-which-the-taylor-polynomial-p-nx-will-converge-to-fx%23new-answer', 'question_page');
    }
    );

    Post as a guest















    Required, but never shown

























    1 Answer
    1






    active

    oldest

    votes








    1 Answer
    1






    active

    oldest

    votes









    active

    oldest

    votes






    active

    oldest

    votes









    1












    $begingroup$

    One such condition is given by Bernstein's theorem: if $(forall xin D_f)(forall ninmathbb{N}):f^{(n)}(x)geqslant0$, then the Taylor series of $f$ converges pointwise to $f(x)$ in the neighborhood of every point of $D_f$.






    share|cite|improve this answer









    $endgroup$


















      1












      $begingroup$

      One such condition is given by Bernstein's theorem: if $(forall xin D_f)(forall ninmathbb{N}):f^{(n)}(x)geqslant0$, then the Taylor series of $f$ converges pointwise to $f(x)$ in the neighborhood of every point of $D_f$.






      share|cite|improve this answer









      $endgroup$
















        1












        1








        1





        $begingroup$

        One such condition is given by Bernstein's theorem: if $(forall xin D_f)(forall ninmathbb{N}):f^{(n)}(x)geqslant0$, then the Taylor series of $f$ converges pointwise to $f(x)$ in the neighborhood of every point of $D_f$.






        share|cite|improve this answer









        $endgroup$



        One such condition is given by Bernstein's theorem: if $(forall xin D_f)(forall ninmathbb{N}):f^{(n)}(x)geqslant0$, then the Taylor series of $f$ converges pointwise to $f(x)$ in the neighborhood of every point of $D_f$.







        share|cite|improve this answer












        share|cite|improve this answer



        share|cite|improve this answer










        answered Dec 5 '18 at 11:35









        José Carlos SantosJosé Carlos Santos

        164k22132235




        164k22132235






























            draft saved

            draft discarded




















































            Thanks for contributing an answer to Mathematics Stack Exchange!


            • Please be sure to answer the question. Provide details and share your research!

            But avoid



            • Asking for help, clarification, or responding to other answers.

            • Making statements based on opinion; back them up with references or personal experience.


            Use MathJax to format equations. MathJax reference.


            To learn more, see our tips on writing great answers.




            draft saved


            draft discarded














            StackExchange.ready(
            function () {
            StackExchange.openid.initPostLogin('.new-post-login', 'https%3a%2f%2fmath.stackexchange.com%2fquestions%2f3026940%2fthe-conditions-under-which-the-taylor-polynomial-p-nx-will-converge-to-fx%23new-answer', 'question_page');
            }
            );

            Post as a guest















            Required, but never shown





















































            Required, but never shown














            Required, but never shown












            Required, but never shown







            Required, but never shown

































            Required, but never shown














            Required, but never shown












            Required, but never shown







            Required, but never shown







            Popular posts from this blog

            Biblatex bibliography style without URLs when DOI exists (in Overleaf with Zotero bibliography)

            ComboBox Display Member on multiple fields

            Is it possible to collect Nectar points via Trainline?